surface chemistry

37
Page No: 69 SURFACE CHEMISTRY Adsorption Phenomenon of attracting and retaining molecules of a substance on surface of liquid or solid resulting in a higher concentration at a surface of solid than in the bulk is called adsorption. The solid substance on the surface of which, adsorption takes place is called adsorbent. While the substance which gets adsorbed on the solid surface due to molecular attractions is called adsorbate.Adsorbent may be solid or liquid but adsorbate may be a gas or a solute in some solution. It is fast in beginnings but become slow after some time. It is exothermic e.g. adsorption of water on silica gel. Adsorption is due to the presence of unbalanced forces, believed to have developed either during crystallisation of solids or due to presence of unpaired, electrons or free valencies in solids having d– orbital. In liquids it is due to surface tension. Desorption It is the reverse of adsorption i.e. removal of adsorbed substance from the surface of adsorbent. This phenomenon is most common in gases adsorbed on solid. Absorption When molecule of a substance are uniformly distributed throughout the body of another substance at uniform rate is called absorption. e.g. absorption of water by CaCl 2 , NH 3 in water. It is assimilation of molecules into a solid or a liquid substance with the formation of a solution or a compound. Sorption Both adsorption and absorption take place simultaneously. e.g. dyes get adsorbed as well as absorbed on the cotton fibers. Difference between adsorption and absorption Sr. No. Adsorption Absorption 1 Adsorption is a surface phenomenon. The adsorbing substance is called adsorbate and is only concentrated on the surface of adsorbent. It is a bulk phenomenon. In absorption the substance penetrates into the bulk of the other substance. 2 The rate of adsorption is rapid to start with and its rate slowly decreases. Absorption occurs at a uniform rate Heat (Enthalpy) of Adsorption Adsorption is a surface phenomenon and adsorbate molecules are held to the surface of adsorbent due to attractive interactions. Since energy is always released during attractive interactions, adsorption is an exothermic reaction. The amount of heat evolved when one mole of an adsorbate gets adsorbed on the surface of an adsorbate is called molar heat (enthalpy) of adsorption. Adsorption in Terms of Gibb’s Helmholtz Equation Adsorption is an exothermic reaction. Therefore, adsorption is accompanied by release of energy or H is always negative and favours the process. Also absorbate molecules get lesser opportunity to move about on the surface of adsorbent. Thus, entropy factor opposes the process. According to Gibb’s Helmholtz equation, G = H – T S Since adsorption does actually take place, H is greater than T S G is negative. As the adsorption continues, the difference between the two opposing tendencies becomes lesser and lesser till they are equal i.e., H

Upload: ashima-aggarwal

Post on 14-Apr-2017

2.119 views

Category:

Education


2 download

TRANSCRIPT

Page 1: Surface  chemistry

Page No: 69

SURFACE CHEMISTRY

Adsorption Phenomenon of attracting and retaining molecules of a substance on surface of liquid or solid resulting in a higher concentration at a surface of solid than in the bulk is called adsorption. The solid substance on the surface of which, adsorption takes place is called adsorbent. While the substance which gets adsorbed on the solid surface due to molecular attractions is called adsorbate.Adsorbent may be solid or liquid but adsorbate may be a gas or a solute in some solution. It is fast in beginnings but become slow after some time. It is exothermic e.g. adsorption of water on silica gel. Adsorption is due to the presence of unbalanced forces, believed to have developed either during crystallisation of solids or due to presence of unpaired, electrons or free valencies in solids having d–orbital. In liquids it is due to surface tension. Desorption It is the reverse of adsorption i.e. removal of adsorbed substance from the surface of adsorbent. This phenomenon is most common in gases adsorbed on solid. Absorption When molecule of a substance are uniformly distributed throughout the body of another substance at uniform rate is called absorption. e.g. absorption of water by CaCl2, NH3 in water. It is assimilation of molecules into a solid or a liquid substance with the formation of a solution or a compound. Sorption Both adsorption and absorption take place simultaneously. e.g. dyes get adsorbed as well as absorbed on the cotton fibers. Difference between adsorption and absorption

Sr. No. Adsorption Absorption 1 Adsorption is a surface phenomenon. The adsorbing

substance is called adsorbate and is only concentrated on the surface of adsorbent.

It is a bulk phenomenon. In absorption the substance penetrates into the bulk of the other substance.

2 The rate of adsorption is rapid to start with and its rate slowly decreases.

Absorption occurs at a uniform rate

Heat (Enthalpy) of Adsorption Adsorption is a surface phenomenon and adsorbate molecules are held to the surface of adsorbent due to attractive interactions. Since energy is always released during attractive interactions, adsorption is an exothermic reaction. The amount of heat evolved when one mole of an adsorbate gets adsorbed on the surface of an adsorbate is called molar heat (enthalpy) of adsorption. Adsorption in Terms of Gibb’s Helmholtz Equation Adsorption is an exothermic reaction. Therefore, adsorption is accompanied by release of energy or H is always negative and favours the process. Also absorbate molecules get lesser opportunity to move about on the surface of adsorbent. Thus, entropy factor opposes the process. According to Gibb’s Helmholtz equation,

G = H – T S Since adsorption does actually take place, H is greater than T S ∵G is negative. As the adsorption continues, the difference between the two opposing tendencies becomes lesser and lesser till they are equal i.e., H = TS or G = 0. At this stage, equilibrium called adsorption equilibrium gets established and there is no net adsorption taking place at this stage. Types of Adsorption There are two types of adsorption.a) Physical adsorption (or Vander waal’s adsorption) or physiosorption.b) Chemisorption a) Physical Adsorption:

In this type of adsorption, molecules of adsorbate are being held to the solid surface by weak attractive forces called Vander Waal’s adsorption. Adsorption of gases like hydrogen or oxygen on charcoal is an example of physical adsorption. It displays the features of physical process such as condensation.

Characteristics of Physical and Chemisorption

Physio–sorption Chemisorption1. Adsorption by weak vanderwaal’s forces. By chemical force (covalent or ionic bond)2. Multimolecular layer may be formed Unimolecular layer will be formed 3. Low heat of adsorption viz. about 20–40

kJ/molHigh heat of adsorption viz. about (200–400 kJ/mol)

4. Easily reversed Not reversed 5. Molecular state of adsorbate on

adsorbent is same. No surface Molecular state may be different. Surface compounds are found.

Page 2: Surface  chemistry

Page No: 70

compounds are found.6. Usually occurs rapidly at low

temperature and decreases with increase in temperature.

It occurs at high temperature initially but then decreases.

7. It increases with pressure Change in pressure will have less effect on chemisorptions

8. Not specific Highly specific 9. Extent of adsorption depends on ease of

adsorption depends on ease of liquefaction i.e. surface area, critical temperature, inversion temperature Vanderwaal’s constant ‘a’, e.g. Adsorption of gases on charcoal

There is no relative between extent of adsorption and ease of liquefaction of gas. e.g. Adsorption of H2 on Pt; decomposition of NH3 in presence of tungsten, decomposition of Hl on gold (zero order)

The difference between physio–sorption and chemisorptions is explained by example of N 2 on Iron. At 83 K it is physio–sorbed on Iron surface as nitrogen molecule. At room temperature there is no adsorption. AT 773 K and above it is chemisorbed on iron surface in the form of nitrogen atom. Positive and Negative Adsorption When the concentration of the adsorbate is more on the surface of the adsorbent than in the bulk, it is called positive adsorption. If the concentration of the adsorbate is less at surface relative to its concentration in the bulk, it is called negative adsorption. e.g. when a concentrated solution of KCl is shaken with blood charcoal, it shows positive adsorption but with a dilute solution of KCl, it shows negative adsorption.

Factor Affecting Adsorption of Gases on SolidsNature of Adsorbent Greater the surface area of adsorbent, greater is the volume of gas adsorbed thus silica gel,aluminium oxide and clay are best adsorbents. Transition metals act as good adsorbents for gases due to vacant or half–filled d–orbitals and high charge – size ratio. Activated charcoal is a better absorbent. Specific Area of Adsorbent Specific area of an adsorbent is the surface area available for adsorption per gram of the adsorbent. Highly porous substances like silica gel, charcoal are very good adsorbents since they have larger surface area. Finely divided substances have large adsorption power. Nature of Adsorbate Easily liquefiable gases (with higher critical temperature) like NH3, HCl, CO2 etc. are adsorbed to a much greater extent than permanent gases like N2, O2, H2 etc. Because easily liqufiable gases have stronger intermolecular forces.e.g. 1 gm activated charcoal adsorbs more SO2 (critical temperature 630 K) then CH4

(critical temperatures, 190 K) Pressure At constant temperature, if pressure is increased, adsorption increases. The increase in much greater if temperature is low.Freundlich gave the relationship between extent of adsorption and pressure. Temperature Adsorption is an exothermic process having an equilibrium: According to Le–Chatelier’s principle, the magnitude of adsorption should increase with decrease in temperature. Gas (Adsorbate) + Solid (Adsorbent) Gas adsorbed + heat The chemisorptions first increases with temperature then decreases. The initial increase shows that like chemical reaction, chemisorptions needs activation energy. Activation of Solid Adsorbent It means increasing the adsorbing power of an absorbent. This is usually done by increasing the surface area of the adsorbent which can be achieved in any of the following ways: (a) By making the surface of the absorbent rough. (b) By subdividing the adsorbent into smaller pieces or grains. (c) By removing the gases already adsorbed Adsorption Isotherms The amount of a gas adsorbed by a given amount of the adsorbent depends upon both temperature and pressure. The variation of adsorption with pressure at a constant temperature is generally expressed graphically. The curves obtained by plotting the amount of gas adsorbed (a = x/m) against gas pressure at a constant temperature are called adsorption isotherms. Adsorption isotherm is a graph between quantities adsorbed under a constant gas pressure at different temperatures. Adsorption isostereis the plot of temperature versus pressure for a given amount of adsorption. Freundlich Adsorption Isotherm for Physisorption

Page 3: Surface  chemistry

Page No: 71

Amount of the gas adsorbed per unit mass of adsorbent increase linearly with pressure in the initial stages and then much more slowly attaining a limiting value as the surface becomes fully covered by gas molecules. The relationship between the magnitude of adsorption and pressure can be expressed

mathematically by an equation:

xm

= kp1/n (n > 1)

The above relationship is commonly known as Freundlich adsorption isotherm. In this relation, x/m is the amount of gas adsorbed per gram of the adsorbent at the pressure p, and ‘k’ and ‘n’ are the constants depending upon the nature of the gas and adsorbent.

The quantity 1/n is generally less than one. This indicates that the amount of the gas adsorbed increases less rapidly than the pressure.

At the lower values of pressure, the graph is nearly a straight line.

xm

∝ P1

At the equilibrium pressure or the saturation pressure (ps), x/m reaches its maximum value, i.e., no more

adsorption takes place even if the pressure is further increased

xm

∝ P0.

At intermediate pressure

xm

∝ P1n

∴ xm

= k P1n

By taking logarithm, the above equation becomes

log xm

= log k + 1nlog P

Thus, if we plot a graph between log (x/m) and log p, a straight line is obtained thus the Frenudlich isotherm is valid. The slope of the line is equal to 1/n and ntercept on log (x/m) axis will correspond to log k. Freundlich isotherm explains the behaviour of adsorption in an approximate manner. The factor 1/n can have taken value between 0 and 1 (probable range 0.1 to 0.5). Thus, the above equation holds good over a limited range of pressure.

When

1n

= 0 , xm

=constant, the adsorption is independent of pressure.

When

1n

= 1 , xm

= k P , i .e . xm

∝ P , the adsorption varies directly with pressure.

Both the conditions are supported by experimental results. The experimental isotherms always seem to approach saturation at high pressure. This cannot be explained by Freundlich isotherm. Thus, it fails at high pressure. Adsorption from Solution Phase Solids can adsorb solutes from solutions also. When a solution of acetic acid in water is shaken with charcoal, a part of the acid is adsorbed by the charcoal and the concentration of the acid decreases in the solution. It has been observed that the extent of adsorption: i) Decreases with an increase in temperature ii) Increases with an increase of surface area of the adsorbent. iii) Depends on the concentration of the solute in solution. iv) Depends on the nature of the adsorbent and the adsorbate.

The precise mechanism of adsorption from solution is not known. Freundlich’s equation approximately

describes the behaviour of adsorption from solution with a difference that instead of pressure,

concentration of the solution is taken into account, i.e.,

xm

= k C1/n

Application of Adsorption 1. In gas mask:

Finally divided coconut charcoal is used as gas masks for absorbing toxic gases like CH 4, CO, COCl2. It is usually used for breathing in coal mines to adsorb poisonous gases.

2. In preserving Vacuum: In Dewar flasks, activated charcoal is placed between the walls of the flask so that any gas which enters into annular space either due to glass imperfection of diffusion through glass is adsorbed.

Page 4: Surface  chemistry

Page No: 72

3. Colloidal silica to purify petroleum oil, motor spirit waxes. 4. Fe(OH)3antidot adsorbent in Arsenic poisoning. 5. In clarification of sugar: Sugar decolorized by treating sugar solution with charcoal powder. The

latter adsorbs the undesirable colours present. 6. Heterogeneous catalysis: Adsorption of reactants on the solid surface of the catalysts increases

the rate of reaction. There are many gaseous reactions of industrial importance involving solid catalysts. Manufacture of ammonia using iron as a catalyst, manufacture of H 2SO4 by contact process and use of finely divided nickel in the hydrogenation of oils are excellent examples of heterogeneous catalysis.

7. In Chromatography: The different chromatographic techniques such as adsorption, paper or column chromatography which are used for the purification and the separation of the substances available in small amounts, are based upon the theory of selective adsorption.

Catalysis Catalyst is a substance which changes the speed of a reaction, and usually, can be recovered completely unchanged at the end of a reaction. However it may take part in a reaction consumed in one step and regenerated in another. This phenomenon is known as Catalysis. Example: Potassium chlorate, when heated strongly decomposes slowly giving dioxygen. The decomposition occurs in the temperature range of 653–873 K.

2KClO3 2KCl + 3O2

However, when a little of manganese dioxide is added, the decomposition takes place at a considerably lower temperature range, i.e., 473–633 K and also at also at a much accelerated rate. The added manganese dioxide remains unchanged with respect to its mass and composition. A substance is termed a positive catalyst or simply as catalyst if it accelerates the rate of chemical reaction. On the other hand, the added substance is termed as negative catalyst if it retards the rate of a chemical reaction. Example of Positive Catalysts: (1) Lead chamber process of H2SO4 using catalyst No.

e.g. 2SO2(g) + O2(g) N⃗O 2SO3(g)

Mechanism O2(g) + 2NO(g) 2NO2(g)

SO2 + NO2(g) SO3(g) + NO(g)

(2) 2KClO3M⃗nO2 2KCl + 3O2

(3) 2CO + O2N⃗O 2CO2

(4) H2O2P⃗t ( s )2H2O + O2

(5) Hydrolysis of ester in acidic medium. Activity of a catalyst refers to the ability of a catalyst to accelerate chemical reaction. e.g. Pt acts as a catalyst in the reaction.

H2(g) + 1/2O2(g)p⃗latinum H2O(l)

Contact process of H2SO4, Where V2O5(s) is used to convert SO2 to SO3. Example of Negative Catalysts: i) H2SO4 or acetanilide in the decomposition of H2O2.ii) Alcohol in the oxidation of chloroform leading to the formation of phosgene. iii) Tertraethyllead or nickel carbonyl acting as antiknock material in internal combustion engines.iv) Antifreezes like glycerol which retard the rusting of the machines. Activation Energy in Catalysed Reaction A catalyst generally increases the rate of reaction. Its behaviour can be explained on the basis of activation energy. According to this concept, a catalyst taking part in a reaction is in a position to lower the height of the energy barrier for the reaction. In fact, the reaction is supposed to follow an alternate path in the presence of catalyst. As a result, the amount of activation energy for the catalysed reaction decreases and the height of the energy barrier gets lowered. This results in increasing the reaction rate or reaction velocity. In a reversible reaction, a catalyst lowers the height of the barrier for both the forward and the backward reactions to the same extent. Therefore, equilibrium point is not distributed, though the equilibrium is reached earlier. Catalytic Promoters There are certain substances which when added only in small quantity to a catalyst enhance its activity. This substance itself may not be catalyst. Such substances which enhance the activity of a catalyst are called catalytic promoters. E.g. Molybdenum is used as Promoter for Fe catalyst in Haber’s process. Catalytic Inhibitors The rates of some reactions are reduced considerably by the presence of small amounts of other substances called inhibitors. For example, the oxidation of sodium sulphide by oxygen gas is inhibited by small amounts of alcohol, aniline and benzaldehyde. Thus, inhibitor is a substance which when added

Page 5: Surface  chemistry

Page No: 73

during the preparation of a catalyst in small amounts, reduces the catalytic activity to a considerable extent. The substance whose presence decreases or destroy the activity of a catalyst are called poison. CO or H2S in hydrogen gas acts as a poison for Fe catalyst in Haber process. As2O3 acts as a poison for Pt–asbestos in contact process for H2SO4. Theories of Catalysis Intermediate Compound Formation Theory This theory explains homogeneous catalysis mainly. According to this theory, the catalyst combines with one of the reactants to give an intermediate compound. This compound intermediately reacts with the other reactants and given the product and regenerates the catalyst in its original form. Thus the reactants do not directly combine with each other, instead they react through the catalyst which provides an alternative pathway which involves lesser energy of activation. For example, the function of nitric oxide [NO] as a catalyst in the formation of SO3 is explained as follows:

Adsorption Theory: This theory explains the heterogeneous catalysis. The role of a solid catalyst in enhancing the reaction rate is explained on the basis of this theory in the following steps: (i) The reactant molecules are adsorbed on the surface of the catalyst at adjacent point. Adsorption

leads to higher concentration of the adsorbed reactant on the surface of a catalyst. (ii) As adsorption is an exothermic process, the heat of adsorption provides the necessary activation

energy for the chemical reaction to proceed and enhance rate of greater. (iii) The product molecules rapidly leave the catalyst surface to make room for the other reactant

molecules to get adsorbed. Thus the chemical combination between reactant molecules occurs at the surface of the catalyst at a must faster rate. e.g. Hydrogenation of ethane in presence of Ni

H – H + 2M 2M – H C2H4 + 2M – H C2H6 + 2M

Modern Adsorption Theory of Heterogeneous Catalysis The modern adsorption theory is the combination of intermediate compound formation theory and the old adsorption theory. The catalytic activity is localized on the surface of the catalyst. The mechanism involves five steps:

Adsorption of reacting molecules, formation of intermediate and desorption of products

i) Diffusion of reactants to the surface of the catalyst. ii) Adsorption of reactant molecules on the surface of the catalyst. iii) Occurrence of chemical reaction on the catalyst’s surface through formation of an intermediate. iv) Desorption of reaction products from the catalyst surface, and thereby, making the surface

available again for more reaction to occur. v) Diffusion of reaction products away from the catalyst’s surface. The surface of the catalyst unlike

the inner part of the bulk, has free valencies which provide the basis for chemical forces of attraction. When a gas comes in contact with such a surface, its molecules are held up there due to loose chemical combination. If different molecules are adsorbed side by side, they may react with each other resulting in the formation of new molecules. Thus, formed molecules may evaporate leaving the surface for the fresh reactant molecules. This theory explain why the catalyst remains unchanged in mass and chemical composition at the end of the reaction and is effective even in small quantities. It however, does not explain the action of catalytic promoters and catalytic poisons.

General Characteristics of Catalytic Reactions1. The catalyst is unchanged chemically at the end of the reaction:

The amount of catalyst remains chemically unaffected at the end of a reaction, though there may be change in physical state such as the particle size or change in the colour of the catalyst.

2. Only a small quantity of the catalysts in generally needed:

Page 6: Surface  chemistry

Page No: 74

Since the catalyst is not used up, a very small amount of catalyst is required. For example, hydrogen peroxide can be decomposed by presence of colloidal platinum at a concentration of 10–

8moldm–3 or by a very small quantity of enzyme catalyst. 3. The catalyst does not alter the position of equilibrium in a reversible reaction

Presence of a small amount of a catalyst does not affect the position of equilibrium. 4. The catalyst does not initiate the reaction

A catalyst simply accelerates or retards a reaction. It does not initiate a reaction. However, this is not true in all the reactions. Many reactions are known to occur only in the presence oif a catalyst.

5. The catalyst is generally specific in its action A catalyst can catalyse only a specific reaction and cannot be used for every reaction. for example, manganese dioxide can catalyse the decomposition of potassium chlorate but not that of potassium nitrate or potassium per chlorate. However, transition metals can catalyse reactions of different types.

6. The catalyst cannot alter the nature of the products of the reaction Potassium chlorate on decomposition, in presence as well as in absence of manganese dioxide, gives oxygen.

7. Optimum temperatures A catalyst has an optimum temperature at which the efficiency of the catalyst is maximum.

8. A catalyst is poisoned by certain substances Presence of traces of certain substances such as arsenious oxide, carbon monoxide, hydrogen cyanide, etc. retards or inhibits the rate of a catalysed reaction to a large extent. Such substances which retard the activity of catalysts are known as catalystic poisons.

9. The activity of a catalyst is enhanced by the presence of substances called promoters:

Types of Catalysis Broadly, two types of catalysis are known: (a) Homogeneous catalysis (b) Heterogeneous catalysis (a) Homogeneous catalysis

If the catalyst is present in the same phase as the reactants, it is called a homogeneous catalyst and this type of catalysis is called homogeneous catalysis.

i) Catalytic decomposition of ozone by chlorine atoms in the gas phase. O3 + O C⃗l 2O2

2CO( g ) + O2( g ) N⃗O 2CO2 ( g )

C12H 22O11 (aq ) + H2O( l) H⃗ 2SO4 C6H 12O6( aq ) + C6H 12O6(aq )Sucrose¿

Lead chamber process of H2SO4 using catalyst NO (all reactant and products are gases) ii) Oxidation of sulphur dioxide into sulphur trioxide with dioxygen in the presence of oxides of

nitrogen as the catalyst in the lead chamber process.

2SO2( g ) + O2 (g ) N⃗O (g ) 2 SO3 (g )iii) Hydrolysis of methyl acetate is catalysed by H+ ions furnished by hydrochloric acid.

CH 3COOCH 3(ℓ ) + H2O( ℓ ) H⃗Cl ( ℓ ) CH 3COOH (aq ) + CH 3OH ( aq )b) Heterogeneous catalysis:

In this type of catalysis the catalyst is present in a difference phase than that of the reactants. In heterogeneous catalysis, catalyst is generally a solid and the reactants are generally gases but sometimes liquid reactants are also used. It is also known as surface catalysis. Many reactions that occur on a metal surface such as the decomposition of Hl on gold and the decomposition of N 2O on platinum, are zero order because the rate determining step occurs on the surface itself.Thus, despite an enormous surface area, once the reactant gas covers the surface, increasing the reactant concentration cannot increase the rate. The petroleum, plastics and food industries frequently use catalytic hydrogenation. The catalyzed reaction is believed to take place through the following consecutive steps.

i) Chemical adsorption of reactants (C2H4, H2) onto the surface of metals. ii) H2 splits into H atoms which get chemically bound to the solid catalyst i.e. metal atom

(M) H–H(g) + 2M(s) 2M – H iii) The H atoms migrate over the surface of the metal and eventually collide with an adsorbed C2H4

molecule and the reaction takes place. C2H4(g) + 2M – H C2H6(g) + 2M(s)

iv) Synthesis of methyl alcohol (CH3OH) from CO and H2 using a mixture of copper, ZnO and Cr2O3 as catalyst.

CO(g) + 2H2(g) C⃗u , ZnO − Cr2O3CH3OH(l)v) Manufacture of ammonia from N2 and H2 by Haber’s process using iron as catalyst.

N2(g) + 3H2(g) 2NH3(g)Important feature of solid catalysts (a) Activity

Page 7: Surface  chemistry

Page No: 75

It is ability of a catalyst to catalyse a process. The activity of a catalyst depends upon the strength of chemisorption to a large extent. The reactants must get adsorbed reasonably strongly on to the catalyst to become active. However, they must not get adsorbed so strongly that they are immobilized and other reactants are left with no space on the catalyst’s surface for adsorption. It has been found that for hydrogenation reaction, the catalytic activity increases from Group 5 to Group 11 metals with maximum activity being shown by group 7–9 elements of the periodic table.

2H2( g ) + O2( g ) P⃗t 2H2O(ℓ )

(b) Selectivity The selectivity of a catalyst is its ability to direct a reaction to yield a particular product. For example, starting with H2 and CO using different catalysts we get different products.

i) CO(g) + 2H2(g) N⃗i CH4(g) + H2O(g)

ii) CO(g) + 2H2(g) C⃗u/ ZnO Cr2O3CH3OH(g)

iii) CO(g) + H2(g) C⃗u HCHO (g) Hydrogenation of alkyne with Ni + H2 or Lindlar catalyst give different product. Action of a catalyst is highly specific (selective) in nature i.e., a given substance can act as a catalyst only a in a particular reaction and not for all the reactions. It means a substance which acts as a catalyst in one reaction may fail to catalyse other reaction i.e., a catalyst is highly selective in nature.

Shape – Selective Catalysis by Zeolites The catalytic reaction that depends upon the pore structure of the catalyst and the size of the reactant and product molecules is called shape selective catalysis. The size of the pores generally varies between 260 pm and 740 pm. Thus only those molecules can be adsorbed in these pores whose size is small enough to enter these cavities and also leave easily. It will not adsorb those molecule which are too big to enter. Thus they act as molecular sieves e.g.: Sodium aluminium silicate can adsorb straight chain hydrocarbons and not branched chain or aromatic ones. The reactions taking place in zeolites depend upon the size and shape of reactant and product molecules as well as upon the pores and cavities of the zeolites. That is why these types of reactions are called ‘shape – selective catalysis’ reaction. Zeolites are being very widely used as catalysts in petrochemical industries for cracking of hydrocarbons and isomerisation. Enzymes are complex nitrogenous compounds which are produced by living plants & animals. Selectivity of a catalyst refers to the ability of a catalyst to direct reaction to yield a particular product (excluding others), e.g. n–heptanes selectively gives toluene in presence of platinum as catalyst. The catalysis that depends upon the pore–structure of the catalyst and molecular sizes of reactants and product molecules is called shape selective catalysis. e.g. zeolites are shape selective catalysts due to their honey–comb structure. ZSM–5 is used for converting methanol to gasoline. Zeolites are micro–porous aluminosilicates of the general formula Mx/n [(AlO2)x (SiO2)y]. zH2O. where n is the charge on the metal cation Mn+ which is usually Na+, K+ or Ca2+ and z is the number of water molecules of hydration which are highly variable. They are three dimensional network silicates in which some silicon atoms are replaced by Al giving Al – O – Si frame work. They exist in nature as well as synthesized in laboratory. They form an important class of oxide catalyst. Zeolites to be used as a catalyst are heated in vacuum so that the water of hydration is lost and they become porous. Enzymes All biological reactions are catalysed by special catalysts called enzymes. Thus, enzymes are defined as biological catalysts. Chemically, all enzymes are globular proteins with high molar mass ranging from 15,000 to 1,000,000g mol–1 and form colloidal solution in water. However, some enzymes are also associated with some non–protein component called the prosthetic group. The prosthetic groups which get attached covalently with enzyme molecule are known as cofactor& the prosthetic group which get attached at the time of reaction are coenzymes. Enzymes are vital for biological processes. Without them, the life process would be very slow and sluggish. For example, if there were no enzymes in our digestive system, it would take us 50 years to digest a single meal. Many enzymes can be obtained in pure crystalline state from living cells. Properties of Enzymes: Some important properties of enzymes are: (i) Specificity

Each enzyme catalyses only one chemical reaction. For example, the enzyme urease hydrolyses urea to NH3 and CO2 but it does not catalyse the hydrolysis of N–methylurea which is similar in constitution to urea.

(ii) Efficiency Enzymes are very efficient catalysts. They speed up rate of a reaction by factors of upto 1020.

(iii) Small quantity Only small amounts of enzymes can be highly efficient.

(iv) Optimum temperature and pHEnzyme catalysed reactions is maximum at particular pH called optimum pH, which is between 5–7 and temperature of 298–310K under one atmospheric pressure. Under the these conditions, most of the chemical reactions do not occur at appreciable rates if ordinary laboratory catalysts are used. Human body temperature being 310 K is suited for enzyme catalysed reactions.

(v) Enzyme activators (Co–enzymes)

Page 8: Surface  chemistry

Page No: 76

The activity of certain enzymes is increased in the presence of certain substances called co–enzymes. e.g. if a protein contains a small amount of vitamin as the non–protein part, its activity is enhanced. The activators are generally metal ions like Na+, Cu+2, Mn+2. Co+2.Amylase in presence of sodium ions are catalytically very active.

(vi) Influence of Inhibitors and poisons: Like ordinary catalysts, enzymes are also inhibited or poisoned by the presence of certain substances. The inhibitors or poisons interact with active functional groups on the enzyme surface and often reduce or completely destroy the catalytic activity of the enzymes. The use of many drugs is related to their action as enzyme inhibitors in the body. The action of enzymes is controlled by a number of mechanisms and are inhibited by certain organic molecules called inhibitors.

Mechanism of Enzyme Catalysis There are a number of cavities present on the surface of colloidal particles of enzymes. These cavities are of characteristics shape and posses active groups such as – NH2, –COOH, –SH-, –OH, etc. These are actually the active centres on the surface of enzyme particles. The molecules of the reactant (substrate), which have complementary shape, fit into these cavities just like a key fits into a lock. On account of the presence of active groups, an activated complex is formed which then decomposes to yield the products.Thus, the enzyme–catalysed reactions may be considered to proceed in two steps. Step 1: Binding of enzyme to substrate to form an activated complex.

E + S ES*Step 2: Decomposition of the activated complex to form product.

ES* E + P

Mechanism of enzyme catalysed reaction Colloidal State

The study for this state of matter was initiated by Thomas Graham in 1861. Substances whose solutions could pass through filter paper and animal membrane, higher rate of

diffusion are called crystalloids. Substances whose solutions are heterogeneous but looks homogenous can pass through filter

paper but not animal membrane, also, having slower rate of diffusion are called colloids. The term colloid does not apply to a particular class of substance but is a state of matter like

solid, liquid and gas. Any substance can be brought into colloidal state by suitable means. Mixtures of substances in water, which can neither pass through filter paper nor animal

membrane are called suspensions. If the solute particles in solutions are in the size range of 1Å to 10Å, the such particles in solution

are called cyrstalloidal particles and the solutions of crystalloidal particles are called true solutions. These are clear solutions. These solutions are homogeneous solutions. It is not possible to separate them by filteration using filter paper or semipermiable membrane.

If the solute particles in solution are of the size in the range of 10 Å to 1000Å, they are called colloidal particles and their solutions are called colloidal solutions.

In suspensions the solute particles are bigger than 1000Å. Such particles are visible with naked eyes and due to their greater volume and greater mass have tendency to settle down under gravity or on long standing.

A colloid is a heterogenous system in which one substance is dispersed (dispersed phase) as very fine particles in another substance called dispersion medium.

Comparison of Suspensions Colloids and True SolutionsS.No. Property True Solution Colloids Suspension

(i) Particle size < 10Å 10Å to 103 Å > 103Å (ii) Visibility Not visible with any of

the optical means Visible with ultramicroscope

Visible with naked eye

(iii) Separation (a) with filter paper(b) with membranes

Not possibleNot possible

Not possible Possible

Possible Possible

(iv) Diffusion Diffuses rapidly Diffuses very slowly does not settle

Does not diffuse

(v) Settling Does not settle but it may settle under Centrifuge

Settles under gravity

(vi) Nature Hemogeneous heterogeneous Heterogeneous (vii) Appearance Clear Generally clear Opaque

Colloids and their Classification

Page 9: Surface  chemistry

Page No: 77

A colloidal system is made of two phases. The substance distributed as the colloidal particles is called Dispersed phase (the internal phase) or the discontinuous phase. The second continuous phase in which the colloidal particles are dispersed is called dispersion medium. For example, for a colloidal solution of copper in water, copper particle constitute the dispersed phase and water the dispersion medium. They are larger molecules with a diameter range between 1 – 1000 nm (10–9 to 10–6 m), but are small enough to remain suspended.Colloidal particles have enormous surface area per unit mass as a result of their small size. Consider a cube with 1 cm side with an surface area 6 cm2. If the were divided into 1012 cubes, the cubes would be the size of large colloidal particles and have a total surface area of 60,000 cm2 or 6 m2. This enormous surface area leads to some special properties of colloids. Classification of Colloids Colloids are classified on the basis of the following criteria: (i) Physical state of dispersed phase and dispersion medium (ii) Nature of interaction between dispersed phase and dispersion medium. (iii) Type of particle of the dispersed phase. (i) Classification Based on Physical State of Dispersed Phase and Dispersion Medium

Depending on the physical states of dispersed phase or dispersion medium, colloidal solutions are of eight types:

Dispersed phase

Dispersion Medium

(appearance) Name (e.g.) alloys

Solid Solid (solid) Solid Sol Coloured glasses, Pearl, Ruby, alloys, gems

Solid Liquid (liquid) Sol Ag, Sol, Au, Sol., S. Sol., Muddy water, gelatin in water, paint

Solid Gas (gas) Aerosol Smoke, dust, stromLiquid Liquid (liquid) Emulsion Milk, medicines, oil water, bloodLiquid Solid (solid) Gel shampoo, jelly, cheese, butter, all

fruits and veg, polish, curdLiquid Gas (gas) Aerosol of liquid Cloud, fog, mist, sprayGas Liquid (liquid) Foam Soap leather, whipped cream,

shaving cream, soda water, froathGas Solid (solid) Solid foam Styrene foam, Foam rubber A colloidal dispersion of one gas in another is not possible since the two gases would give a homogenous molecular structure. Many familiar commercial products and natural objects are colloids. For example, whipped cream is a foam, which is a gas dispersed in a liquid. Firefighting foams, used at emergency aeroplane landings are also colloidal systems. Most biological fluids are aqueous sols (solids dispersed in water). Within a typical cell, proteins and nucleic acids are colloidal–sized particles dispersed in an aqueous solution of ions and small molecules. Out of the various types of colloidal given in table the most common are sols (solids in liquids), gels (liquids in solids) and emulsions (liquids in liquids). If the dispersion medium is water, the sol is called aquasol or hydrosol and if the dispersion medium is alcohol, it is called alcosoland so on. (ii) Classification Based on Nature of Interaction between Dispersed Phase and Dispersion

Medium Depending upon the nature of interaction between the dispersed phase and the dispersion medium, colloidal sols are divided into two categories, namely, lyophilic (solvent attracting) and lyophobic (solvent repelling). If water is the dispersion medium, the terms used are hydrophilic and hydrophobic.

(a) Lyophilic Sols Colloidal solutions in which the dispersed phase has considerable affinity for the dispersion medium, are called lyophilic sols (solvent–linking). For example – dispersion of gelatin starch, gum and proteins in water. Such colloidal solutions can be easily prepared in water and in general and called emulsoids. These solutions are stable known as reversible colloids since the residue left on evaporating can be readily transferred back into colloidal state by adding water. These sols are quite stable and cannot be easily coagulated.

(b) Lyophobic Sols Colloidal solution in which the dispersed phase has no affinity or attraction for the dispersion medium are called Lyophobic colloid (solvent hating) solutions. Colloidal solutions of metals which have negligible affinity for solvents are examples of this type. Lyophobic colloidal solutions are less stable. On evaporation of solvent the residue cannot be easily transferred back into colloidal state by ordinary mean. Therefore, lyophobic colloids are also called irreversible colloids. Lyophobic sols need stabilizing agents for their preservation. Such sols are readily precipitated on addition of small amount of electrolytes by heating or by shaking.

Comparison of Lyophobic and Lyophilic sols

SN Property Lyophobic sol (Suspensoid) Lyophilic sol (Emulsoid) 1 Preparation Cannot be prepared easily, special

methods are required Can be easily prepared by shaking or warming the substance with solvent

Page 10: Surface  chemistry

Page No: 78

2 Stability are less stable are more stable 3 Reversibility are irreversible are reversible 4 Viscosity Viscosity is nearly same as that of

the solvent Viscosity is much higher than that of the solvent

5 Surface tension Surface tension is almost same as that of solvent

Surface tension is usually low

6 Hydration or solvation These are less solvated as the particles have less affinity for the solvent

These are highly solvated as the particles have great affinity for solvent

7 Charge The particles carry a characteristic charge either positive or negative

The particles have little charge or no charge at all

8 Visibility Particles can be seen under microscope

Particles cannot be seen under microscope

9 Coagulation or precipitation

Precipitated by low concentration of electrolytes

Precipitated by high concentration of electrolytes

10 Tyndall effect Tyndall effect is well marked due to appreciable difference between refractive index of dispersion phase and medium.

As particles are much solvated so difference between the refractive index of 2 phases is very less. So tyndall effect is not well marked.

11 Migration on applyingelectric field

Migrate towards anode or cathode as these carry charge.

May or may not migrate.

12 General example Mostly inorganic nature. Mostly organic nature

(iii) Classification Based on Type of Particles of the Dispersed Phase Depending upon the type of the particles of the dispersed phase, colloids are classified as:

(a) Multimolecular Colloids: On dissolution, a large number of atoms or smaller molecules of a substance aggregate together to form species having size in the colloidal range (diameter < 1 nm). The species thus formed are called multimolecularcolloids. For example, a gold sol may contain particles of various sizes having many atoms. Sulphur sol consists of particles containing a thousand or more of S8sulphur molecules.

(b) Macromolecular ColloidsThese are formed by macromolecules which have bigger size than the colloidal particle but as soon as they are put in suitable solvents, they get dissociated to form smaller particle of the colloidal range. Such systems are called macromolecular colloids. These colloids are quite stable and resemble true solutions in many respects. Examples of naturally occurring macromolecules are starch, cellulose, proteins and enzymes; and those of man–made macromolecules are polythene, nylon, polystyrene, synthetic rubber, etc.

(c) Associated Colloids (Micelles) There are some substances which at low concentrations behave as normal strong electrolytes, but at higher concentrations exhibit colloidal behaviour due to the formation of aggregates. The aggregated particles thus formed are called micelles. These are also known as associated colloids. The formation of micelles takes place only above a particular temperature called krafttemperature (Tk) and above a particular concentration called critical micelle concentration (CMC). Conductance of solution decreases at CMC. Shape of micelles changes with change in concentration. e.g. at high concentration rod shaped micelles are formed while spherical micelles are formed near CMC. On dilution, these colloids revert back to individual ions. The importance of micelles and their use is based on the fact that their hydrophobic interior can dissolve fat or oil etc. while the water soluble part, makes a hydrophilic surface around this interior, rendering the entire micelle water soluble. Surface active agents such as soaps and synthetic detergents belong to this class. For soaps, the CMC is 10–4 to 10–3mol L–1. These colloids have both lyophobic and lyophilic parts. Micelles may contain as many as 100 molecules or more.

Mechanism of Micelle Formation Let us take an example of soap solutions. Soap is sodium or potassium salt of a higher fatty acid and may be represented as RCOO Na+ (e.g., sodium stearate CH3(CH2)16COO–Na+, which is a major component of many bar soaps). When dissolved in water, it dissociates into RCOO– and Na+ ions. The RCOO– ions, however, consist of two parts – a long hydrocarbon chain R (also called non – polar ‘tail’) which is hydrophobic (water repelling), and a polar group COO– (also called polar–ionic ‘head’), which is hydrophilic (water loving).

Page 11: Surface  chemistry

Page No: 79

Hydrophobic tail and hydrophilic head of stearate ion

The RCOO– ions are, therefore, present on the surface with their COO– groups in water and the hydrocarbon chains R staying away from it and remain at the surface. But at critical micelle concentration, the anions are pulled into the bulk of the solution and aggregate to form a spherical shape with their hydrocarbon chains pointing towards the centre of the sphere with COO– part remaining outward on the surface of the sphere. An aggregate thus formed is known as ‘ionic micelle’. These micelles may contains as many as 100 such ions.

Similarly, in case of detergents. e.g., sodium laurylsulphate, CH3(CH2)11 OSO3−Na+ , the polar group is

−OSO3−

along with the long hydrocarbon chain. Hence, the mechanism of micelle formation here also is same as that of soaps. Note: The soaps and detergents form micelles in water only because of the presence of charge on their molecules. Micelles formation does not occur in solvent like ethyl alcohol since it is not as polar as soaps. That is why only water is used for the washing of dirty clothes.

Method of Preparation of Colloidal Solution Lyophilic sols may be prepared by simply warming the solid with liquid dispersion medium e.g., starch with water. On the other hand lyophobic sols have to be prepared by special methods. These methods fall into two categories:

(a) Dispersion methods in which large macrosized particles are broken down to colloidal size.

(b) Condensation methods in which colloidal sized particles are built up by aggregating single ions or molecules. This method is also known as condensation method.

Sr. No.

Dispersion method Aggregation or condensation method

1 Mechanical dispersion 1. Exchange of solvents 2 Electro–dispersion 2. Change of physical state3 Ultrasonic dispersion 3. Chemical method 4 Peptization (a) Double decomposition

(b) Oxidation (c) Reduction (d) Hydrolysis

(a) Dispersion Method 1. Mechanical dispersion (e.g. black ink, paint, varnish) dyeSolid material is first finely grounded by usual methods. It is then mixed with dispersion medium which gives a coarse suspension. The suspension is now introduced into the colloid mill. The simplest form of colloid mill consist of two metal discs held at a small distance apart from one another and capable of revolving at a very high speed (about 7000

Colloidal mill

Page 12: Surface  chemistry

Page No: 80

revolutions per minute) in opposite directions. The particles are grounded down to colloidal size and are then dispersed in liquid. A stabilizer is often added to stabilize colloidal solution.

Colloidal graphite (a lubricant) and printing ink are made by this method. Tannin is used as a stabilizer in the preparation of colloidal graphite and gum Arabic in lampblack colloidal solution (Indian ink)

2. Electrical Dispersion (Bredig’s arc Method) (e.g. metal sol.) This method is suitable for the preparation of colloidal solutions of metals like gold, silver, platinum, etc. An arc is struck between the metal electrodes under the surface of water containing some stabilizing agent such as a trace of KOH. The water is cooled by immersing the container in a ice bath. The intense heat of the arc vaporizes some of the metal which condenses under cold water.

Note: (i) This method is not suitable when the dispersion medium is an organic liquid as considerable

charring occurs. (ii) This method comprises both dispersion and condensation. 3. Peptization

The dispersion of a freshly precipitated material into colloidal solution by the action of an electrolyte in solution is termed peptization,the electrolyte used is called a peptizing agent. During Peptization, the precipitate adsorbs one of the ions of the electrolyte on its surface. This causes the development of positive or negative charge on precipitates, which ultimately break up into smaller particles of the size of a colloid.

Few examples of sols obtained by peptization are: (i) Freshly prepared ferric hydroxide on treatment with a small amount of ferric chloride

solution at once forms, a dark raddish brown solution. Ferric chloride acts as a peptizing agent.

(ii) Freshly precipitated silver chloride can be converted into a colloidal solution by adding a small amount of hydrochloric acid or AgNO3

(iii) Cadmium sulphide can be peptized with the help of hydrogen sulphide. H2S is used as peptizing agent for most of the sulphide sol.

4 Hydrolysis Colloidal solutions of some salts can be prepared by hydrolysis. A colloidal solution of ferric hydroxide is obtained by boiling a dilute solution of ferric chloride.

FeCl3 + 3 H2O Fe(OH)3 + 3HClred sol.

The colloidal solutions of silicic acid is also obtained by hydrolysis of dilute solution of sodium silicate with 4N hydrochloric acid which is added drop by drop with constant stirring.

Purification of Colloidal Solutions Colloidal solutions prepared by above methods generally contain excessive amount of electrolytes and some other impurities. The purification of colloidal solution is carried out by the following methods:

Dialysis: Animal membranes (bladder) or those made of parchment paper and cellophane sheet have very fine pores. These process permit ions or very small molecules to pass through but not large colloidal particles. Dialysis is a process of removing a dissolved substance (impurities) from a colloidal solution by means of diffusions through semipermiable which is a bag of suitable membrane containing colloidal solution to be purified, placed in a vessel (or continuous flow of water). The ions or molecules of impurities diffuse through membrane and get dissolved in outer water and pure colloidal solution is left in the bag. Blood is a colloidal solution and is purified by dialysis.

The process of dialysis is a slow process. It can be made faster by applying an electric field. Two electrodes are placed in water outside the membrane bag containing the colloidal solution. When potential difference is applied across the membrane, ions in the solution move faster towards opposite electrode. This process is called electrodialysis. The colloidal solution is placed in a bag of suitable membrane while pure water is taken outside. Electrodes are fitted in the compartment as shown in figure.

Colloidal mill

Page 13: Surface  chemistry

Page No: 81

The ions present in the colloidal solution migrate out towards the oppositely charged electrodes.

Ultra–filtration: Ultra–filtration is the process of separating colloidal particles from the solvent and soluble solute in the colloidal solution by especially prepared filters, which are permeable to all substances except colloidal particles. Such filter are called ultrafilters. e.g. a cellophane membrane. Colloidal particles can pass through ordinary filter paper because the pores are too large. However, the pores of filter paper can be reduced in size by impregnating with colloidion solution to stop the flow of colloidal particles. The usual colloidion is a 4 % solution of nitro–cellulose in a mixture of alcohol and ether. An ultra–filter paper may be prepared by soaking the filter paper in a colloidal solution, hardening by formaldehyde and then finally drying it.Ultra–filtration is a slow process. To speed up the process, pressure or suction is used. The colloidal particles are left on the ultrafilter in the form of slime. This slime may be stirred into fresh medium to get a pure sol.

Ultracentrifugation: In this method, the impure sol is taken in a tube which is placed in an ultracentrifuge. In this machine, the tube is rotated at a very high speed. As a result, the colloidal particles settle down at the bottom of the tube whereas the crystalloids and other soluble impurities remain in the solution. This solution is decanted off and the colloid particles are remixed with the dispersion medium to give the pure colloidal sol.

Properties of Colloidal Solution Heterogeneous:

Colloidal particles in a solution differ in sizes and are not homogeneously distributed throughout the solution.

Visibility: Colloidal particles cannot be seen with naked eyes or with the help of microscope. It is a well known fact. No particle is visible if its diameter is less than half the wavelength of light used. The visible light has greater wavelength than the size of colloidal particle.

Colour: The colour of hydrophobic sol depends on the wavelength of the light scattered by the dispersed particles. The wavelength of the scattered light again depends on the size and the nature of particles. The colour of colloidal solution also changes with the manner in which the observer receives the light. For example, a mixture of milk and water appears blue when viewed by reflected light and red when viewed by the transmitted light. Finest gold sol is red in colour, as the size of particles increases, it appears purple, then blue and finally golden. When light emitted by the setting sun passes through the blanket of dust, the blue part of the light is scattered away from out eyes and at the same time the red colour is seen. This sun appears red while setting.

Colour of Ag Sol Particle diameter Orange Yellow 6 × 10–5 mmOrange Red 9 × 10–5 mmPurple 13 × 10–5 mmViolet 15 × 10–5 mm

Colligative Properties These properties depend on the number of solute particles in solution. In case of colloidal solutions, colloidal particles are the aggregates of many ions or smaller molecules and when compared to true solutions or normal solutions, the total number of particles of solute in solution are very less and hence these solutions exhibit colligative properties to lesser extent.

Optical Properties: Tyndalll effect Sols exhibit Tyndall effect. When a beam of light is passed through a sol and viewed at right angles.

The path of the light shows up a hazy beam of cone. This was first observed by Farraday and later by Tyndall and is known as Tyndall effect. It may be defined as the scattering of light by the colloidal particles in a colloidal sol. The bright cone of the light is called “Tyndall cone”. The Tyndall effect is due to the fact that the colloidal particles absorb light and scatter it in all colloidal dispersion. The phenomenon is also observed when a beam of light is projected in a cinema hall and it become visible due to the scattering by colloidal dust particles in the air of the room. Tyndall effect is observed only when the following two conditions are satisfied (i) The diameter of the dispersed particles is not much smaller than the wavelength of light used. (ii) The refractive index of dispersed phase & the dispersion medium differ greatly in magnitude.

Some example of Tyndall effect are: (i) Blue colour of sky and sea water. (ii) Visibility of tails of comets.

Page 14: Surface  chemistry

Page No: 82

(iii) Twinkling of stars.The importance of this effect lies in the fact that it has helped to confirm the heterogeneous nature of the colloidal solutions and is used to distinguish between a colloidal and true solution. Zsigmondy, in 1903, used Tyndall effect to set up an apparatus known as ultramicro–scope. An intense beam of light is focused on the colloidal solution contained in a glass vessel. the focus of the light is then observed with a microscope at right angles to the beam. Individual colloidal particles appear as bright starts against a dark background. Ultra micro–scope does not render the actual colloidal particle visibility but only observe the light scattered by them. Thus ultra microscope does not provide any information about the size and shape of colloidal particles. Kinetic Properties: (Brownian Movement)

When a sol is examined with an ultramicroscope, the suspended particles are seen as shining speeks of light. By following an individual particle, it is observed that the particle is in a state of continuous motion in zig–zag directions. The continuous rapid zig–zag motion of a colloidal particle in the dispersion medium is called “Brownian movement of motion” (first observed by British botanish Robert Brown).

The Brownian movement has been explained to be due to the unbalanced bombardments of the particles by the molecules of dispersion medium. This motion is independent of the nature of the colloid but depend on the size of the particles and viscosity of the solution. Smaller the size and lesser the viscosity, faster is the motion. The Brownian movement has a stirring effect which does not permit the particles to settle and thus, is responsible for the stability of sols. Charge on Colloidal Particles:

Colloidal particles always carry an electric charge. The mutual forces of repulsion between similarly charged particles prevent them from aggregating and settling under the action of gravity. This gives stability to the sol. A list of common sols with the type of charge on their particles is given below.

SN Positively Charged Negatively Charged 1. Hydrated metallic oxides

e.g.: Al2O3.xH2O, CrO3.xH2O and Fe2O3.xH2O etcMetals e.g. copper, silver, gold sols.

2. Basic dye stuff example – Methylene blue sol Metallic sulphides like As2S3, Sb2S3, CdS3. Proteins in acidic medium Hemoglobin (blood) Acid dye stuff example – Congored sols,

eosin, albumin.4. Oxides like TiO2, etc Sols of starch, gum gelatin, clay & charcoal

The charge on colloidal particles is due to one or more of the following reasons: (i) The sol particles acquire positive or –ve charge by preferential adsorption of +ve or –ve ions from

the dispersion medium. Preferential adsorption of ions is the most accepted reason. The sol particles acquire positive or negative charge by preferential adsorption of +ve or –ve ions. When two or more ions are present in the dispersion medium, preferential adsorption of the ion common to the colloidal particle usually takes place. This can be explained by taking the following examples: (a) When silver nitrate solution is added to potassium iodide solution, the precipitated silver iodide

absorbs iodide ions from the dispersion medium and negatively charged colloidal solution results. However, when Kl solution is added to AgNO3 solution, positively charged sol results due to adsorption of Ag+ ions from dispersion medium.

Agl/l– Agl/Ag+

Negatively charged Positively charged (b) If FeCl3 is added to excess of hot water, a positively charged sol of hydrated ferric oxide is formed

due to adsorption of Fe3+ ions. However, when ferric chloride is added to NaOH a negatively charged sol is obtained with adsorption of OH– ions.

Fe2O3.xH2O/Fe3+ Fe2O3.xH2O/OH–

Positively charged Negatively charged Having acquired a positive or a negative charge by selective adsorption on the surface of a colloidal particle as stated above, this layer attracts counter ions from the medium forming a second layer, as shown below.

Agl/l– K+ Agl/Ag+ l–

The combination of the two layers of opposite charges around the colloidal particle is called Halmholtz electrical double layer. According to modern views, the first layer of ions is firmly held and is termed fixed layer while the second layer is mobile which is termed diffused layer. Since separation of charge is a basis of potential, the charges of opposite signs on the fixed and diffused parts of the double layer results in a difference in

Page 15: Surface  chemistry

Page No: 83

potential between these layers. This potential difference between the fixed layer and the diffused layer of opposite charges is called the electrokinetic potential or zeta potential. The presence of equal and similar charges on colloidal particle is largely responsible in providing stability to the colloidal solution, because the repulsive forces between charged particles having same charge prevent them from coagulating or aggregating when they come closer to one another.

Electrical Properties: (a) Stability of colloidal sols – Electrical charge on colloidal particles: The dispersed phase

particles carry either +ve and –ve charge and dispersion medium has an equal and opposite charge. The particles repel one another and hence do not coagulate, thus making the colloid stable. Origin of electrical charge on colloidal particles: a. Due to frictional electrification b. due to electron capture by sol particles. c. Due to preferential adsorption of ions from the solution. d. Due to dissociation of molecule electrolytes adsorbed on the surface of the particles. e. Due to dissociation of molecules followed by aggregation of ions

(b) Cataphoresis or Electrophoresis is the movement of colloidal particles either towards cathode or anode, depending on their charge, under the influence of an electric field. Electrophoresis is used to determine nature of charge Positive sol. Sol Fe(OH)3, Ca(OH)2, Al(OH)3, basic dye stuffs (methylene blue, methyl violet, Haemoglobin).Negative Sol. Metal &sulphides: CdS, As2S3, Cu, Ag, Pt and dyes (congored, Prussian blue, silicic acid, gum, charcoal).

(c) Electro Osmosis: A sol is electrically neutral therefore the dispersion medium carries an equal but opposite charge to that of dispersed particles. Thus the medium will move in opposite direction to the dispersed phase under the influence of applied electric potential if colloidal solution is enclosed within semipermeable membrane. The movement of dispersion medium under the influence of applied potential is known as “Electro–osmosis.” Electro–osmosis is direct consequence of the existence of zeta potential between the sol particle and the medium. When the applied potential exceeds the Zeta potential, the diffused layer move and causes electro–osmosis.

(d) Coagulation or flocculation: [Process of setting of colloidal particle] We known that the stability of a lyophobic sol is due to the adsorption of positive or negative ions by the dispersed particles. The repulsion forces between the charged particles do not allow them to settle. If somehow, the charge is removed there is nothing to kept the particles apart from each other. In such cases they aggregate or flocculate and settle down under the action of gravity.The flocculation and setting down of the discharged sol particles is called coagulation or the precipitation and can be brought about in following ways: (a) By addition of electrolyte. (b) By electrophoresis (c) By mixing two oppositely charged sols. (mutual precipitation) (d) By boiling (e) Persistent dialysis.

(a) By Addition of Electrolytes: When an electrolyte is added in excess to a sol, the electrolyte furnishes both types of ions in solution. The oppositely charged ions get adsorbed on the surface of colloidal particles. This causes neutralization and thereby the size and mass of colloidal particle increases and it becomes a suspension particle. Due to greater volume and greater mass these suspension particles settle down i.e., they coagulate. The ion irresponsible for neutralization of charge on the particle is called flocculating ion.

Coagulation or flocculation is the process of bringing colloidal particles closer so that they aggregate to form larger particle that precipitate and settle down or float on the surface. It is usually done by addition of an electrolyte. Effective ions or electrolyte are those which carry charge opposite to colloids.

Hardy – Schulze rule states that “greater is the valency of the oppositely charged ion of electrolyte being added, faster is the coagulation: e.g. for a negatively charged sol, the order is: Al3+> Ba2+> K+,

for a positively charged sol the order is: Fe(CN )6

4− > PO43− > SO4

2− > Cl−

Page 16: Surface  chemistry

Page No: 84

Coagulating value is the minimum amount of electrolyte in milli moles required to coagulate a 1 lit of solution of sol in two hours. The smaller the quantity needed, the higher will be the coagulating power of anion.

(b) By Electrophoresis:During electrophoresis the charged sol particles migrate towards the electrode of opposite sign. There they deposit their charge and then get coagulated (As the neutral particles can aggregate and change to suspension particles).

(c) By Mixing Two Oppositely Charged Sols: The mutual coagulation of two sols of opposite charge can be effected by mixing them. For e.g. Fe(OH)3 (positive sol) and Arsenioussulphide (negative sol) when mixed join and coagulate.

(d) By Boiling Sols such as sulphur and silver halides disperse in water, get coagulated when boiled, due to increased collisions between sol particles and water molecules, which remove the adsorbed layer from the sol and therefore the sol particles settle down.

(e) Persistent dialysis: The stability of a colloidal sol is due to presence of a small amount of the electrolyte. On prolonged dialysis, the electrolyte is completely removed. As a result, the colloidal sol becomes unstable and gets coagulated.

Coagulation of Lyophillic Sols: Lyophilic sols are stable due to charge and solvation of the colloidal particles. When these two factors are removed, a lyophilic sol can be coagulated. This is done (i) by adding electrolyte; (ii) by adding suitable solvent. When solvents such as alcohol and acetone are added to hydrophilic sol, the dehydration of dispersed phase occurs. Under this condition a small quantity of electrolyte can bring about coagulation. Protection of Colloids: Lyophilic sols are more stable than lyophobic sols hence they

are used as protective colloids to increase the stability of lyophobic sols, e.g. addition of gums, gelatin etc. to certain metal sols. These sols are called Protective colloids.

Protective action of lyophilic sols is explained due to formation of a thin layer around the lyopobic sol particles, thus preventing them from coming closer.

Gold number is a term used to compare protective action of different lyophilic colloids.

Gold number of a lyophilic sol is the minimum amount of it in milligram, which prevent the coagulation of 10ml gold sol (no colour change from red to blue) against 1 ml of 10% NaCl solution.

The precipitate of gold sol is indicated by a colour change from red to blue when the particle size just increases.

Higher the gold number, lesser is the protective power. Gelatin has highest and starch has lowest protective ability.

Emulsions Emulsion is a colloidal system involving one liquid dispersed in another, provided both are immiscible. One of the constituent is oil while the other is water. Normally these are not expected to be stable because oil and water do not mix. These are therefore stabilized by the presence of certain substances called emulsifier or emulsifying agents. Some of the emulsifying agents are soaps, agar, gum etc. the emulsifying agent forms and interfacial film between suspended particles and the medium. Some commonly known emulsions are Milk, butter, milk cream, cold cream, vanishing cream, etc.

There are many drugs and medicines which are also in the form of emulsions e.g. various ointments, cod liver oil etc.

Types of emulsions: Depending upon the type of proportions in which these constituents are present, emulsion have been classified into two types.

1. Oil in water emulsions (O/W) are those in which oil is the dispersed phase and water is the dispersion medium. Milk is a common example in which liquid fats are dispersed in water. Similarly, if a few drops of nitrobenzene (oil) is added to water, an emulsion results. Varnishing cream is another example of this type.

2. Water in oil emulsions (W/O) are those in which water is the dispersed phase and oil is the dispersion medium, Butter is an example of water in oil emulsion in which water is dispersed in oil. Cod liver oil and cream are the other examples of these emulsions.

Page 17: Surface  chemistry

Page No: 85

Detection of the types of Emulsions The nature of an emulsion whether O/W or W/O can be detected with the help of following tests.

(a) Dilution test. To the emulsion taken in a beaker, add a small amount of water. In cases it gets diluted, it is an example of oil in water type emulsion. If water forms a separate layer then the emulsion is water in oil type. Milk, for example, can be readily diluted with water. It is therefore, oil in water type emulsion.

(b) Dye test. Add a small amount of oil soluble dye to the emulsion. In case, only a small portion of it gets coloured, it is an example of oil in water emulsion. However, if the entire background is coloured, it is an example of water in oil emulsion.

Emulsifier and Emulsifying Agents An emulsifier or emulsifying agent is a substance which helps in stabilizing an emulsion of oil and water and, thus, prevents them from getting apart. The commonly used emulsifying agents for O/W emulsion are soaps, detergents, lyphilic colloids, proteins, gums and agar etc. & for W/O emulsion are heavy metal salt of long chain fatty acids etc. The preparation of emulsion in the presence of an emulsifier is known emulsification. The role of an emulsifier in stabilizing an emulsion can be explained in two ways:

(a) It is believed that an emulsifier gets concentrated at the oil–water interface i.e., the surface at which oil and water come in contact with each other. It forms a protective coating around each drop of oil and thus, prevents the oil drops from coming in contract with one another. The oil drops remain suspended in water and are not coagulated.

(b) According to an another view, the role of the emulsifier is the same as that of lubricant in a machine. Just as a lubricant reduces he friction in the various parts of machine, an emulsifier also tries to reduce the interfacial tension between oil and water by suitable means. Thus, oil and water remain in company of each other and do not get separated. The cleaning action of soap in washing dirty clothes illustrates the role of emulsifier.

Application of emulsions The important applications of emulsions in daily life are as follows: (i) The concentration of the sulphide ore of a metal by froth floatation process involves the use of

some oil such as pine oil. The oil forms emulsion with ore particles. When air is bubbled through the emulsion, it rises to the surface as foam and is skimmed off.

(ii) The various drugs available in liquid form such as cod liver oil, B–complex, etc., are emulsions of water in oil type. These are readily adsorbed in the intestines. Digestive enzymes act on emulsified drugs in order to carry out their metabolic functions.

(iii) The cleansing action of soap is based upon the formation of oil–in–water type emulsion. (iv) Milk which is an important constituent of our diet is an emulsion of fat in water. (v) Cosmetics, lotions, creams and many ointments are emulsions. Several oily drugs are prepared as

emulsions in order to facilitate their adsorption. Demulsification

It is the process of decomposing an emulsion back into its constituent liquids. The demulsification can be done by centrifugation, filtration, boiling, freezing and by some chemical methods such as change in pH. For example, centrifugation separates cream (fat) from milk which is an emulsion. When water is the dispersion medium, the presence of a dehydrating agent can result in the demulsification.

Nature of an emulsion can be distinguished by dilution, viscosity and conductivity tests. Emulsions show all the properties of colloidal sols. The process of breaking an emulsion to yield the constituent liquids is known as demulsification

e.g. heating, freezing, centrifuge. Emulsions are useful as several pharmaceutical preparations, cosmetics, dairy products etc. Droplet of emulsion are often negatively charged and can be precipitated by electrolytes

Gels These are colloidal systems where liquids are the dispersed phase and solids act as dispersion

medium, e.g. curd, cheese etc. When a warm solution of gelatin is cooled it sets to a semisolid mass which is gel. This process is called gelation.

Applications of Colloidal System

Page 18: Surface  chemistry

Page No: 86

Colloids are widely used in the industry. Following are some examples. 1. Medicines in colloidal form can be easily adsorbed by body tissues & hence are more effective e.g.

colloidal antimony is used in curing kalaazar, argyrols is a silver sol used as an eye lotion. Colloidal gold is used for intramuscular injection Neutralization of excess acidity by Al (OH)3, Milk of magnesia, an emulsion, is used for stomach disorders, colloidal sulphur – germ killer, kalolin used to remove toxins from intestine. Cod liver oil is emulsion of oil in water. Some ointment, antibiotics, Penicillin, streptomycin are produced in colloidal form. Colloidal medicines are more effective because they have large surface area and are therefore easily assimilated.

2. Food: Gelatin in ice–cream. It preserve smoothness and prevent ice crystals formation, protein, milk, cheese are colloids, fruits, jelly, cream, bread is dispersion of air in baked dough.

4. Purification of water by alums: Water from rivers or lakes are sometimes used for domestic and industrial purpose after purification. The water from lake or river is turbid due to the presence of fine clay particles which are negatively charged. These can be removed by adding potash alum or aluminiumsulphate. Al3+ ions from potash alum o aluminiumsulphate neutralize the negative charge on clay particles. This causes the coagulation of clay particles which settle down leaving water which is clear of further treatment.

5. Stoppage of bleeding from a fresh cut: The bleeding from a fresh cut can be immediately stopped by applying a concentrated solution of ferric chloride or potash alum. Blood consists of colloidal particles of haemoglobins which carry positive charge on them. When ferric chloride or alum is applied on the cut these colloidal particles get their positive charge neutralized by the anions available from these substances in solution. In the absence of the charge, they get coagulated and the bleeding stops.

6. Delta formation: Formation of delta due to coagulation of colloidal particles of river water by sea salt. The river water contains colloidal particles of sand and clay which carry negative charge. The sea water, on the other hand, contains positive ions such as Na+, Mg2+ and Ca++. As the river water meets sea water, these ions discharge the sand or clay particles which are precipitated as delta.

7. Photographic plates and films: Photographic plates or films are prepared by coating an emulsion of the light sensitive silver bromide in gelatin over glass plates or celluloid films.

8. Fog, mist and rain. In extremely cold weather, the temperature is very low. The water droplets (moisture) present in air condense on the surface of the dust particles that are suspended. Since these are colloidal in nature, they float in air in the form of mist or fog or extremely low, these colloidal droplets grow in size. They become bigger and bigger and when atmosphere is not in a position to hold them, they come down as rain. In the winter season, the atmosphere generally becomes very foggy and visibility is quite poor. This leads to lot of inconvenience for vehicular movement and result in major accidents. Farmers are in the habit of burning husk in the open fields. This results in the release of unburnt carbon particles in the atmosphere. The water droplets condense on them and they appear as fog. But actually this is smog and not fog and is extremely injurious to health. This can lead to asthama, lung as well as throat cancer. It is very essential to educate our farmer community about these harzards.

9. Purification of Blood: Blood is a colloidal solution. It contains toxic waste products such as urea and uric acid which pass through the membrane while colloidal sized particles of blood proteins hemoglobin are retained. Kidney failure, therefore, leads to death due to accumulation of poisonous waste products in the blood. Blood is purified by dialysis.

10. Chemical warfare: (Smoke screen): Smoke is a colloidal solution of solid particles such as carbon, arsenic compounds, dust, etc., in air. The smoke, before it comes out from the chimney, is lead through a chamber containing plates having a charge opposite to that carried by smoke particles. The particles on coming in contract with these plates lose their charge and get precipitated. The particles thus settle down on the floor of the chamber. The precipitator is called Cottrell precipitartor. Colloidal dispersions of certain substance, like TiO2 (as dispersed phase) in air (as the dispersion medium) are used a smoke screens in warfare for the purpose of concealment. The dispersed phase particles (i.e. TiO2) being very heavy rapidly drops down in the form of a curtain of drizzling whiteness.

Page 19: Surface  chemistry

Page No: 87

11. Artificial rain: Tiny water droplets in clouds are electrically charged. In any cloud all such water particles carry the same charge. Artificial rain can be caused by spraying oppositely charged dust or fine sand or precipitates like Agl (which has a crystal structure similar to ice and as such particles of Agl can act as nuclei for precipitation) on to a cloud. Even certain salts in t he form of fine particles can be sprayed. The neutralization of charge results in coagulation of water droplets or rain. Cloud–brust–a natural disaster resulting in a very heavy down pour over a very short time is believed to occur due to the mutual discharge of oppositely charged clouds.

12. Cleaning Action of Soaps and Detergents: Dust and dirt particles on clothes are colloidal in nature. Soaps being sodium salts of higher fatty acids coagulate them which become suspension particles called micelle and roll out due to greater volume and greater mass.

13. Rubber Industry: Latex is a colloidal solution of rubber particles which are negatively charged. Rubber is obtained by coagulation of Latex. Rubber is electroplated on metals when they act as anode.

IIT – JEE AssignmentQ.1 The coagulation of 100 cm3 of gold solution so completely prevented by addition of 0.25g of starch

to it before adding 10 ml of 10% NaCl solution. The gold number of starch is

(a) 0.025 (b) 0.25 (c) 2.5 (d) 25

Q.2 For adsorption of a gas on a solid, the plot of log x/m vs log P is linear with slope equal to: (n being a whole number)

(a) k (b) log k (c) n (d) 1/n

Q.3 For the coagulation of 100 mL of As2S3 sol, 5 mL of 1 M NaCl is required. The flocculation value of NaCl is

(a) 50 (b) 5 (c) 47.6 (d) None of these

Q.4 Which of the following colloid can be prepared by electrical dispersion as well as reduction method?

(a) Sulphur (b) Ferric hydroxide (c) Arsenioussulphide (d) Gold

Q.5 Silver iodide is used for producing artificial rains because silver iodide

(a) is easy to spray at high altitude (b) is insoluble in water

(c) has crystal structure similar to ice (d) is easy to synthesize

Answers1. d 2. d 3. a 4. d 5. c

IIT– JEE Assignment Q.1 The particles of a particular colloidal solution of arsenic trisulphide (As2S3) are negatively

charged. Which 0.0005M solution of the following salts would be most effective in coagulating this colloidal solution, KCl, MgCl2, AlCl3 or Na3PO4? Explain.

Q.2 A sample of charcoal weighing 6g was brought into contact with a gas contained in a vessel of one litre capacity at 27oC. The pressure of the gas was found to fall from 700 to 400 mm of Hg. Calculate the volume of the gas (reduced to STP) that is adsorbed per gram of the adsorbent under the condition of the experiment (density of charcoal sample is 1.5 g cm–3).

Q.3 1 gm of active charcoal is taken and its surface area is 3.01 × 102 m2/gm. It adsorbs 100 ml of 0.5 M CH3COOH in a single layer. After adsorption, its molarity becomes 0.49M. Find the surface area of the charcoal covered by one molecule of acetic acid.

Q.4 A solution of palmitic acid (M = 256 g mol–1) in benzene contains 4.24g of acid per dm3. When this solution is dropped on a water surface the benzene evaporates and the palmitic acid forms a monomolecular film of the solid type. If we wish to cover an area of 500 cm 2 with a monolayer, what volume of solution should be used? The area covered by one palmitic acid molecule may be taken to be 0.21 nm2.

Page 20: Surface  chemistry

Page No: 88

Q.5 A one-litre vessel contained a gas at 27oC. 6g of charcoal was introduced into it. The pressure of the gas fell down from 700 mm Hg of 400 mm Hg. Calculate the volume of the gas (at S.T.P.) adsorbed per gram of charcoal. Density of charcoal sample used was 1.5g cm–3.

Q.6 In an adsorption experiment, a graph between log (x/m) versus log p was found to be linear with a slope of 45o. The intercept on the log (x/m) axis was found to be 0.3010. Calculate the amount of the gas adsorbed per gram of charcoal under a pressure of 0.5 atmosphere.

Q.7 The volume of nitrogen gas vm(measured at S.T.P.) required to cover a sample of silica gel with a mono-molecular layer is 129 cm3 g–1 of gel. Calculate the surface area per gram of the gel if each nitrogen molecule occupies 16.2 × 10–20 m2.

Q.8 1 g of charcoal adsorbs 100 ml of 0.5M CH3COOH to form a monolayer, and thereby the molarity of CH3COOH reduces to 0.49. Calculate the surface area of the charcoal adsorbed by each molecule of acetic acid. Surface area of charcoal = 3.01 × 102 m2/g.

Q.9 20% of surface sites are occupied by N2 molecules. The density of surface sites is 6.023 × 1014 cm–2

and total surface area is 1000 cm2. The catalyst is heated to 300 K while N2 is completely desorbed into a pressure of 0.001 atm and volume 2.46 cm3. Find the active sites occupied by each N2

molecule.

Q.10 A particle of radius 1 cm is broken to form colloidal particles of radius 1000Å. Calculate the no. of colloidal particles produced and their total surface area.

Q.11 When 5.19 × 10–5 g of palmitic acid (C15H31COOH) in the form of a dilute solution in benzene was spread on the surface of water, it could be compressed to an area of 265 cm2 before the resisting force increased sharply. Calculate the area occupied by a single molecule in the closely packed layer.

Q.12 Compare the coagulating power of AlCl3 with that of NaCl. Given that their coagulation values are 0.093 and 52 respectively.

Q.13 One gram of activated charcoal has a surface area of 1000 m2. If complete coverage is assumed, how much ammonia (in cm3 at STP) could be adsorbed on the surface of 25g of the charcoal? Given: diameter of NH3 molecule = 0.3 nm.

Q.14 50 mL of standard gold sold needs 0.05 mg of gelatin for its protection from coagulation. Calculate the gold number of gelatine.

Answer

1. AlCl3 2. 60.19 ml g–1 3. 5 × 10–19 m2

4. 0.024 ml 5. 60.168 cm3 6. 1.0

7. 562 m2 8. 5 × 10–19 m2 9. 2

10. 12.57 × 105 cm2 11. 2.17213 × 10–15 cm2 12. 559

13. 13151 cm3 14. 0.01

Objective Assignment

Q.1 Which of the following metal sols cannot be prepared by Bredig’s arc method?

(a) copper (b) potassium (c) gold (d) platinum

Q.2 Gelatin protects

(a) gold sol (b) As2S3 sol (c) Fe(OH)3 sol (d) all of these

Q.3 Flocculation value is expressed in terms of

(a) milli mole/L (b) mole/L (c) grams/L (d) mole/ mL

Q.4 The gold numbers of four protective colloids O, P, Q and R are 0.005, 0.01, 0.1 and 0.5 respectively. The decreasing order of their protective power is

(a) R, Q, P, O (b) O, P, Q, R (c) P, Q, R, O (d) Q, R, O, P

Q.5 A sol has positively charged colloidal particles. Which of the following solutions is required in lowest concentration for coagulation?

(a) NaCl (b) K4[Fe(CN)6] (c) ZnCl2 (d) Na2SO4

Q.6 The protective power of lyophilic sol is

(a) dependent on the size of colloidal particles

(b) expressed in terms of gold number (c) expressed by x/m

(d) directly proportional to the magnitude of charge on it

Q.7 Which plot is the adsorption isobar for chemisorptions?

Q.8 The stability of the dispersed phase in a lyophobic colloid is due to

Page 21: Surface  chemistry

Page No: 89

(a) high viscosity of the medium (b) the formation of electrical layer between two phases

(c) high surface tension of solution (d) none of the answer is correct

Q.9 Tyndall effect is due to

(a) reflection of light (b) scattering of light (c) absorption of light (d) adsorption of light

Q.10 The ability of ion to bring about coagulation of a given colloidal solution depends upon

(a) the size of its ion (b) the magnitude of charge

(c) the sign of charge (d) both magnitude and sign of charge

Q.11 Purple of cassius is

(a) colloidal solution of gold (b) colloidal solution of silver

(c) colloidal solution of platinum (d) oxyacids of gold

Q.12 Which of the following is not a colloid?

(a) chlorophyll (b) smoke (c) Paint (d) milk

Q.13 Gold number is minimum in case of

(a) gelatin (b) egg albumin (c) gum arabic (d) starch

Q.14 Lyophilic sols are more stable than lyophobic sols because

(a) the colloidal particles have positive charge (b) the colloidal particles have negative charge

(c) the colloidal particles are solvated

(d) there are strong electrostatic repulsions between the negatively charged colloidal particles

Q.15 On adding few drops of dil. HCl to freshly precipitated ferric hydroxide, a red coloured colloidal solution is obtained. This phenomenon is known as

(a) peptisation (b) dialysis (c) protective action (d) dissolution

Q.16 Asenioussulphide solution caries a negative charge. The maximum precipitating power of this sol is possessed by

(a) K2SO4 (b) CaCl2 (c) Na3PO4 (d) AlCl3

Q.17 In colloidal state, particles size ranges from

(a) 1 to 10Å (b) 20 to 50 Å (c) 10 to 1000 Å (d) 1 to 280 Å

Q.18 Which one of the following substance gives a positively charged sol?

(a) gold (b) A metal sulphide (c) Ferric hydroxide (d) an acidic dye

Q.19 Peptisation denotes

(a) digestion of food (b) hydrolysis of proteins

(c) breaking and dispersion into colloidal state(d) precipitation of a solid from colloidal state

Q.20 A colloidal solution is subjected to an electrical field. The particles move towards anode. The coagulation of same sol is studied using NaCl, BaCl2 and AlCl3 solutions. Their coagulating power should be

(a) NaCl> BaCl2> AlCl3 (b) BaCl2> AlCl3>NaCl

(c) AlCl3> BaCl2>NaCl (d) BaCl2>NaCl> AlCl3

Q.21 Which of the following is les than zero during adsorption?

(a) G (b) S (c) H (d) all of the above

Q.22 Cellulose dispersed in ethanol is called

(a) emulsion (b) micelle (c) colloidion (d) hydrophilic solution

Q.23 Which of the following is most suitable for the coagulation of ferric hydroxide sol.

(a) KCl (b) KNO3 (c) K2SO4 (d) K3[Fe(CN)6]

Q.24 Which of the following is not a characteristic of chemisorptions?

(a) adsorption is irreversible (b) H is of the order of 900 kJ

(c) adsorption is specific (d) adsorption increases with increase of surface area

Q.25 Alums purify muddy water by:

(a) dialysis (b) adsorption (c) coagulation (d) forming a true solution

IIT–JEE–Single Choice Correct

Q.26 According to Freundlich adsorption isotherm, which of the following is correct?

(a) x/m p1 (b) x/m p1/n (c) x/m po

(d) all the above are correct for different ranges of pressures

Page 22: Surface  chemistry

Page No: 90

Q.27 The critical micellization concentration (CMC) is:

(a) the concentration at which micellization begins

(b) the concentration at which true solution is formed

(c) the concentration at which one molar electrolyte is present per 1000gm of solution

(d) the concentration at which solute and solution form equilibrium

Q.28 Which one of the following is correctly matched?

(a) emulsion – curd (b) foam – mist (c) aerosol– smoke (d) solid sol – cake

Q.29 Adsorption is accompanied by:

(a) decrease in entropy of the system (b) decrease in the enthalpy of the system

(c) decreases in Gibbs free energy (d) all of these

Q.30 Which reaction gives colloidal solution:

(a) Cu + HgCl2 CuCl2 + Hg (b) 2 HNO3 + 3H2S 3S + 4H2O + 2NO

(c) 2Mg + CO2 2MgO + C (d) Cu + CuCl2 Cu2Cl2

Q.31 A freshly prepared Fe(OH)3 precipitate is peptized by adding FeCl3 solution. The charge on the colloidal particle is due to the preferential adsorption of:

(a) Cl– (b) Fe3+ ions (c) OH– ions (d) none of these

Q.32 Which statement is incorrect?

(a) higher the gold number of lyophilic substance better is its protective action

(b) lower the gold number of a lyophilic substance better is its protective action

(c) theBredig’s arc method is usually suitable for preparing sols of metals

(d) the osmotic pressure method gives the average molar mass of a polymer

Q.33 50 mL of 1M oxalic acid (hydrated) is shaken with 0.5g wood charcoal. The final concentration of the solution after adsorption is 0.5 M. What is the amount of oxalic acid adsorbed per gm of carbon?

(a) 3.15 g (b) 3.45 g (c) 6.30 g (d) none of these

Q.34 Colloidol solution of arseniussulphide can be prepared by:

(a) electro dispersion method (b) peptization

(c) double decomposition (d) hydrolysis

Q.35 Ferric chloride is applied to stop bleeding due to a cut because:

(a) Fe3+ ion coagulates blood which is a negatively charged sol

(b) Fe3+ ion coagulated blood which is a positively charged sol

(c) Cl– ion coagulates blood which is a positively charged sol

(d) Cl– ion coagulated blood which is a negatively charged sol

Q.36 Minimum concentration of an electrolyte which is able to cause coagulation of a sol is termed as its:

(a) emulsification value (b) saponification value

(c) flocculation value (d) gold number

Q.37 Which of the following forms cationic micelles above certain concentration?

(a) sodium dodecyl sulphate (b) sodium acetate

(c) urea (d) cetyltrimethyl ammonium bromide

Q.38 Critical micelle concentration (CMC) of soap solutions lies in the range:

(a) 10–6 – 10–5 M (b) 10–5 – 10–4 M (c) 10–4 – 10–3 M (d) 10–3 – 10–2 M

Q.39 Which equation represents Freundulich adsorption isotherm (physical adsorption is basis of this theory)

(a)

xm

= K (P)1/n

where x is amount of gas adsorbed on mass ‘m’ at pressure P

(b) log x

m= log K + 1

nlog P

(c)

xm

= K .P . at low pressure and

xm

= Kat high pressure

(d) all of these

Q.40 The Brownian motion is due to:

(a) temperature fluctuations with in the liquids phase

(b) attraction and repulsion between charges on the colloidal particles

(c) impact of the molecules of dispersion medium on the colloidal particles

(d) convective currents

Q.41 The cotterells precipitator is used to:

Page 23: Surface  chemistry

Page No: 91

(a) neutralize charge on carbon particles in air is smoke

(b) coagulate carbon atoms of smoke

(c) bring in cataphoresis in carbon particles (d) all of these

Q.42 An example of intrinsic colloid is:

(a) As2S3 sol (b) Fe(OH)3 sol (c) egg albumin (d) Au sol

Q.43 Freundlich adsorption isotherm gives straight line on plotting

(a)

xm

vs . P(b) log ( xm ) vs . P

(c)log ( xm )vs . logP

(d)

xmvs . 1

P

Q.44 In homogeneous catalytic reactions, there are three alternative paths A, B and C (shown in the figure). Which one of the following indicates the relative ease with which the reaction can take place?

(a) A > B > C

(b) C > B > A

(c) B > C > A

(d) A = B= C

Q.45 For the reaction (A B + C); the energy profile diagram is given in the figure. The decrease in energy of activation in presence of catalyst is

(a) z

(b) z – p

(c) y – z

(d) z – x

Q.46 The colloidal system consisting of a liquid adsorbate in a solid adsorbent is termed as

(a) aerosol (b) foam (c) emulsion (d) gel

Q.47 Which is false for a catalyst?

(a) a catalyst can initiate a reaction

(b) it does not alter the position of equilibrium in a reversible reaction

(c) a catalyst remains unchanged in quality and composition at the end of reaction

(d) catalysts are very specific in reaction

Q.48 Which of the following statements is incorrect?

(a) adsorption always leads to a decrease in enthalpy and entropy of the system

(b) adsorption arises due to unsaturation of valence forces of atoms or molecules on surface

(c) adsorption increases with rise in temperature

(d) adsorption decreases the surface energy

Q.49 Which of the following gas molecules have maximum value of enthalpy of physio–sorption?

(a) C2H6 (b) Ne (c) H2O (d) H2

Q.50 Which of the following gases is adsorbed most by activated charcoal?

(a) CO2 (b) N2 (c) CH4 (d) Ar

More than One Choice Correct

Q.51 Which of the following statements are correct?

(a) physical adsorption is multilayer, non–directional and non specific

(b) chemical adsorption first increases and than decreases with increase in temperature

(c) insame cases, solvent may be adsorbed in preference to the solute on the surface of the adsorbent

(d) as a result of adsorption, there is increase of surface energy

Q.52 Which of the following are aerosols?

(a) smoke (b) fog (c) milk (d) butter

Q.53 Which of the following are correctly matched?

(a) milk – emulsion (b) butter – gel (c) fog – aerosol (d) dust – solid sol

Q.54 Which of the following increases the activation of a solid adsorbent?

(a) polishing the surface of the solid adsorbent

(b) subdividing the solid adsorbent

Page 24: Surface  chemistry

Page No: 92

(c) blowingsuper heated steam through the porous adsorbent

(d) carrying out the adsorption at very high temperature

Q.55 Which of the following is incorrect about chemisorptions:

(a) multilayer adsorption (b) reversible in nature

(c) strong adsorption by free valences (d) Exothermic in nature

Q.56 Adsorption is accompanied by:

(a) decrease in enthalpy (b) increase in free energy of system

(c) decrease in entropy of system (d) the value of TS is negative

Q.57 Which of the following will give linear plots?

(a) log ( xm )v / s log c

(b)

log ( xm )v / s ( 1p )(c)

(mx ) v /s ( 1p )(d)

p( x / m)

v /s p

Q.58 Zeolites are:

(a) hydratedalumino–silicates which can be used for shape selective catalysis

(b) have general formula Mx/n [(AlO2)x (SiO2)y]. mH2O where, n is charge on metal Cation Mn+.

(c) have pore sizes between 260 pm to 740 pm

(d) ZSM–5 which is used in petroleum industry as a catalyst, converts hydrocarbons into alcohols

. Q.59 Lyophilic sols are characterized by:

(a) colloidal particles are solvated (b) reversible in nature

(c) colloidal particles always have charge (either positive or negative)

(d) smallquanity of electrolyte is required for precipitation

Q.60 Colloidal solutions prepared by chemical processes contains impurities in the form of electrolyte or the soluble substance. Which of the following methods are commonly used to purify a colloidal solution.

(a) peptization (b) eletrodialysis

(c) Bredig’s arc method (d) ultra centrifugation

Q.61 If x/m is the mass of adsorbate adsorbed per unit mass of adsorbent, P is the pressure of the adsorbate gas, a & b are constants, which of the following represent Langmuir adsorption isotherm?

(a) log( xm ) = log ( ab ) + 1

alog P

(b)

xm

= aP1 + bP

(c)

xm

ab (when the pressure is high) (d)

xm

= a .P (when the pressure is low)

Q.62 Which one of the following graphs correctly represents effect of temperature on the process of adsorption?

Q.63 The extent of adsorption of a gas on solid depends on:

(a) nature of the gas (adsorbate) (b) pressure of the gas

(c) temperature of the gas (d) nature of adsorbent

Q.64 The coagulation of sol particles may be brought in by:

(a) adding oppositely charge sol (b) adding electrolyte

(c) heating (d) persistent dialysis

Page 25: Surface  chemistry

Page No: 93

Q.65 With the help of electrophoresis

(a) formation of colloidal sol occurs

(b) coagulation of colloidal solution occurs if carried out for prolonged times

(c) charge on colloidal particles can be predicted

(d) it is possible to separate different proteins, carbohydrates & nucleic acids

Match the following

Q.66 Note: Each statement in column – I has one or more than one match in column – II.

Column – I Column – II

I. Tyndall effect (A) Au, Ag, Pt sol

II. Sluphur sol (B) Oxidation

III. Electrical disintegration (C) Scattering of light

IV. Lyophobic sol (D) Colloidal solution

Q.67

Column – I Column – II

I. Physical adsorption (A) stable

II. Chemical adsorption (B) less stable

III. Gelatin (C) Irreversible

IV. Gold sol (D) reversible

Note:Each statement in column – I has only one match in column – II.

Q.68

Column – I Column – II

I. Purple of cassius (A) congo–rubin dye

II. Langmiur isotherm (high p) (B) x/m = a/b

III. freundlich isotherm (intermediate p) (C) colloidal sol

IV. Rubin number (D) x/m = k.p1/n

(E)

Assertion & Reason Type

Direction: Read the following questions and choose

(A) If both Assertion and Reason are true and Reason is the correct explanation of the Assertion.

(B) If both Assertion and Reason are true but Reason is not correct explanation of the Assertion.

(C) If Assertion is true but Reason is false(D)If Assertion is false but Reason is true

Q.69 Assertion: A sol of As2S3 prepared by the action of the H2S on As2O3 is negatively charged.

Reason: It is due to the adsorption of H+ ions on the surface of colloidal particles and S2– ion get diffused in todispersion medium.

Q.70 Assertion: For As2S3, sol, BaCl2 has higher coagulation value than NaCl

Reason: Higher the valency of the oppositely charged ion of the electrolyte added, higher is the coagulation value of the electrolyte.

Q.71 Assertion: Aqueous gold colloidal solution is red in colour.

Reason: The colour arises due to scattering of light by colloidal gold particles.

Q.72 Assertion: Physical adsorption of molecules on the surface required activation energy.

Reason: The physical adsorption is due to weak Vander Waal force of attraction between adsorbent and adsorbate.

Q.73 Assertion: Haber’s synthesis of NH3 is carried out in the presence of catalyst.

Reason: The catalyst shifts the position of the equilibrium of the reaction.

(a) (A) (b) (B) (c) (C) (d) (D)

Passage Based Problems

A colloid is a heterogeneous system in which one substance is dispersed (dispersed phase) as a very fine particles in another substance called dispersion medium. Depending upon whether the dispersed phase and the dispersion medium are solids, liquids or gases, eight type of colloidal systems are possible. They are prepared in the industry or in the laboratory by a number of methods and then purified. Hardy and &

Page 26: Surface  chemistry

Page No: 94

Schulze made a substantial contribution in studying the coagulation of colloids. The protective power of lyophilic sol was studied by Zsigmondy and he introduced the term ‘Gold number’.

Q.74 For the coagulation of 100 mL of arseniussulphide sol 5 mL of 1 M NaCl is required. The coagulating value of NaCl is

(a) 46.7 (b) 47.6 (c) 49.9 (d) 42.0

Q.75 In an experiment on electro–osmosis, in which of the following the level of the dispersion medium will fall on the cathode side?

(a) gold sol (b) starch sol (c) Fe(OH)3 sol (d) As2S3 sol

Q.76 If dispersed phase is a liquid and the dispersion medium is solid, the colloid is know as

(a) a sol (b) a gel (c) an emulsion (d) a foam

Q.77 Which of the following anions will have minimum flocculation value for ferric hydroxide so?

(a) SO4

2−

(b) Cl– (c) Br– (d) [Fe(CN)6]3–

Answers

1 b 11 a 21 d 31 b 41 d

2 d 12 a 22 c 32 a 42 c

3 a 13 a 23 d 33 c 43 c

4 b 14 c 24 b 34 c 44 b

5 b 15 a 25 c 35 a 45 b

6 b 16 d 26 d 36 c 46 d

7 c 17 c 27 a 37 d 47 a

8 b 18 c 28 c 38 c 48 c

9 b 19 c 29 d 39 d 49 c

10 d 20 c 30 b 40 c 50 a

Answers More than One Option

51 a,b, c, 52 a,b 53 a,b, c 54 b,c 55 a,b

56 a, c,b 57 a, c,d 58 a,b, c 59 a, b 60 b, d

61 b,c, d 62 a, c 63 a,b,c, d 14 a, b,c,d 65 b,c, d

Match the following

66 I–a, c, d ;II–b,c,d ;III–a,c,d ;IV–a,b,c,d

67 I–b,d ;II–a,c ;III–a,d ;IV–b,c

68 I–c ;II–b ;III–d ;IV–a

Assertion & Reason

69 c 70 d 71 a 72 d 73 c

74 b 75 c 76 b 77 d